- Tue Oct 17, 2017 4:42 pm
#40640
Complete Question Explanation
(The complete setup for this game can be found here: lsat/viewtopic.php?t=4425)
The correct answer choice is (E)
The question stem asks you to identify a group of three paralegals who cannot be assigned to Sicoli. Thus, each of the four incorrect answer choices Could Be True, and the one correct answer choice Cannot Be True.
The easiest—and fastest—point of attack is to check for the presence of H, since H must be assigned to Sicoli from the third rule. Alas, it would simply be too easy for the correct answer to be missing H, and thus this approach fails (but it’s always worth a shot because some LSAT questions are just easy; this just doesn’t happen to be one of them). However, in glancing at the answer choices, you should have noted that each answer choice contains three paralegals, so the question is being evaluated under the 3-1-1 distribution, with all three paralegals assigned to Sicoli.
From our discussion of the 3-1-1, in the group of three we noted that exactly one of the three paralegals will be from the group of F and G, and exactly two of the paralegals will be from the group of H, K, and L. H is already assigned to Sicoli, and thus we know the following about the composition of the group of three:
As a follow up, note that answer choices (A), (B), (C), and (D) each contain exactly one of K and L. Answer choice (E) contains both of K and L, again supporting that this answer choice is correct.
(The complete setup for this game can be found here: lsat/viewtopic.php?t=4425)
The correct answer choice is (E)
The question stem asks you to identify a group of three paralegals who cannot be assigned to Sicoli. Thus, each of the four incorrect answer choices Could Be True, and the one correct answer choice Cannot Be True.
The easiest—and fastest—point of attack is to check for the presence of H, since H must be assigned to Sicoli from the third rule. Alas, it would simply be too easy for the correct answer to be missing H, and thus this approach fails (but it’s always worth a shot because some LSAT questions are just easy; this just doesn’t happen to be one of them). However, in glancing at the answer choices, you should have noted that each answer choice contains three paralegals, so the question is being evaluated under the 3-1-1 distribution, with all three paralegals assigned to Sicoli.
From our discussion of the 3-1-1, in the group of three we noted that exactly one of the three paralegals will be from the group of F and G, and exactly two of the paralegals will be from the group of H, K, and L. H is already assigned to Sicoli, and thus we know the following about the composition of the group of three:
- 1 paralegal = H
1 paralegal from the group of F and G = F/G
1 paralegal from the group of K and L = K/L
As a follow up, note that answer choices (A), (B), (C), and (D) each contain exactly one of K and L. Answer choice (E) contains both of K and L, again supporting that this answer choice is correct.